The Boffo Product Company sells a waffle iron on which they have done product testing. They have determined that the amount of time the product will last can be described by a normal distribution. In particular, the average waffle iron lasts for 12 years and one standard deviation is 8 months. How long should they warranty the product for if they want no more than 6.7% of the waffle irons to fail within that time

Answers

Answer 1

Answer:

They should warranty the product for 7 years if they want no more than 6.7% of the waffle irons to fail within that time.

Step-by-step explanation:

Normal Probability Distribution:

Problems of normal distributions can be solved using the z-score formula.

In a set with mean [tex]\mu[/tex] and standard deviation [tex]\sigma[/tex], the z-score of a measure X is given by:

[tex]Z = \frac{X - \mu}{\sigma}[/tex]

The Z-score measures how many standard deviations the measure is from the mean. After finding the Z-score, we look at the z-score table and find the p-value associated with this z-score. This p-value is the probability that the value of the measure is smaller than X, that is, the percentile of X. Subtracting 1 by the p-value, we get the probability that the value of the measure is greater than X.

The average waffle iron lasts for 12 years and one standard deviation is 8 months.

Measuring the time in months, we have that [tex]\mu = 12*8 = 96[/tex] and [tex]\sigma = 8[/tex]

How long should they warranty the product for if they want no more than 6.7% of the waffle irons to fail within that time?

This is X when Z has a p-value of 0.067, so X when Z = -1.5. Then

[tex]Z = \frac{X - \mu}{\sigma}[/tex]

[tex]-1.5 = \frac{X - 96}{8}[/tex]

[tex]X - 96 = -1.5*8[/tex]

[tex]X = 84[/tex]

84 months = 7 years.

They should warranty the product for 7 years if they want no more than 6.7% of the waffle irons to fail within that time.


Related Questions

A team of researchers wants to determine whether pet owners are generally more satisfied with their lives than non-pet owners. To test their theory, the researchers randomly select 500 pet owners and 500 non-pet owners from several major metropolitan areas in the country. The researchers then interview the individuals, asking them a series of questions. Each response is assessed with a point value that is later translated to a satisfaction indicator. Of the pet owners surveyed, 380 of the 500 were found to be satisfied with their lives, while 336 of the 500 non-pet owners were found to be satisfied.
Would this study be considered an experiment or an observational study?

Answers

Answer:

observational study

Step-by-step explanation:

Which of the following expressions have a difference of 5? Check all that apply.
VX
0-3-(-8)
-2-3
1-(-4)
07-(-2)

Answers

9514 1404 393

Answer:

  A, C

Step-by-step explanation:

A: -3 -(-8) = -3 +8 = 5

B: -2 -3 = -5

C: 1 -(-4) = 1 +4 = 5

D: 7 -(-2) = 7 +2 = 9

__

Differences A and C are 5.

Answer: A: -3- (-8)

B:1 -(-4)

Step-by-step explanation:

in A your subtracting them in B your adding them

Ivan runs a cake shop. Renting the
shop costs him $1600 per month,
and he makes a profit of $16 on each
cake he sells. Ivan wants a profit of at
least $2000 a month.

Answers

He’s gotta sell atleast 125 cakes
He’s gotta sell 125 cakes

Find the difference of (4.2x10^3)-(2.7x10^3)
Show work!

Answers

Step-by-step explanation:

Is it helpful ?

plz let me know

How tall is the average human baby ?

Answers

Full term- 20” (50cm)
Normal range 18” to 22”

Assume that a population is normally distributed with a mean of 100 and a standard deviation of 15. Would it be unusual for the mean of a sample of 3 to be 115 or more? Why or why not?

Answers

Answer:

|Z| < 2, which means that it would not be unusual for the mean of a sample of 3 to be 115 or more.

Step-by-step explanation:

To solve this question, we need to understand the normal probability distribution and the central limit theorem.

Normal Probability Distribution:

Problems of normal distributions can be solved using the z-score formula.

In a set with mean [tex]\mu[/tex] and standard deviation [tex]\sigma[/tex], the z-score of a measure X is given by:

[tex]Z = \frac{X - \mu}{\sigma}[/tex]

The Z-score measures how many standard deviations the measure is from the mean. After finding the Z-score, we look at the z-score table and find the p-value associated with this z-score. This p-value is the probability that the value of the measure is smaller than X, that is, the percentile of X. Subtracting 1 by the p-value, we get the probability that the value of the measure is greater than X.

If [tex]|Z| > 2[/tex], the value of X is considered to be unusual.

Central Limit Theorem

The Central Limit Theorem establishes that, for a normally distributed random variable X, with mean [tex]\mu[/tex] and standard deviation [tex]\sigma[/tex], the sampling distribution of the sample means with size n can be approximated to a normal distribution with mean [tex]\mu[/tex] and standard deviation [tex]s = \frac{\sigma}{\sqrt{n}}[/tex].

For a skewed variable, the Central Limit Theorem can also be applied, as long as n is at least 30.

Assume that a population is normally distributed with a mean of 100 and a standard deviation of 15.

This means that [tex]\mu = 100, \sigma = 15[/tex]

Sample of 3

This means that [tex]n = 3, s = \frac{15}{\sqrt{3}}[/tex]

Would it be unusual for the mean of a sample of 3 to be 115 or more? Why or why not?

We have to find the z-score.

[tex]Z = \frac{X - \mu}{\sigma}[/tex]

By the Central Limit Theorem

[tex]Z = \frac{X - \mu}{s}[/tex]

[tex]Z = \frac{115 - 100}{\frac{15}{\sqrt{3}}}[/tex]

[tex]Z = 1.73[/tex]

|Z| < 2, which means that it would not be unusual for the mean of a sample of 3 to be 115 or more.

Which is the graph of f(x) = 2 (4)?
5
40.4)
404)
4
(4,4)
3
3
3 2
2
2
2
(2.1)
6,2)
1
5 -4 -3 -2 -14
1
3
4
-5 4 -3 -2 -14
234
-5 6 -3 -2 -14
2
3
4
5
X
-2
-2
نا دیا
-3
-3
4
W4
-5
5
Tu
5
4
(
24)

Answers

Answer:

The Third one

Step-by-step explanation:

Your Welcome :)

Graph of the function is attached below.

Correct option is D.

What is exponential function?

As the name suggests, the exponential function contains an exponent. Note, however, that the exponential function has a constant as its base and a variable as its exponent, not vice versa (if a function has a variable as its base and a constant as its exponent, it is a power function). The exponential function can be in one of the following forms:

Definition of exponential function

In mathematics, an exponential function is a function of the form f(x) = aˣ. where "x" is a variable and "a" is a constant  called the base of the function, which must be greater than 0.

Given, exponential function

f(x) = (1/4)4ˣ

exponential function is defined for x∈R

Putting x = 0

f(0) = (1/4)4⁰

f(0) = 1/4

Point on curve is (0,1/4)

Putting x = 1

f(1) = (1/4)4¹

f(1) = (1/4)4

f(1) = 1

Point on curve is (1,1)

Putting x = 2

f(2) = (1/4)4²

f(2) = (1/4)16

f(2) = 4

Point on curve is (2,4)

Putting x = 3

f(3) = (1/4)4³

f(3) = (1/4)64

f(3) = 16

Point on curve is (3,16)

Point (0, 1/4), (1, 1), (2, 4), (3, 16) can be used to draw graph of the function.

Hence, graph of the function is drawn as follows.

Learn more about exponential function here:

https://brainly.com/question/14355665

#SPJ7

What translation maps ABC to A'B'C'?

Answers

The answer is D because if u trans it up 4 then C and B won’t be in the right place. So and rotation 90 ccw will do the job.

The Rockwell hardness of a metal is determined by impressing a hardened point into the surface of the metal and then measuring the depth of penetration of the point. Suppose the Rockwell hardness of a particular alloy is normally distributed with mean 70 and standard deviation 3. (Rockwell hardness is measured on a continuous scale.)a. If a specimen is acceptable only if its hardness is between 67 and 75, what is the probability that a randomly chosen specimen has an acceptable hardness?b. If the acceptable range of hardness is (70-c, 70+c) , for what value of c would 95% of all specimens have acceptable hardness?c. If the acceptable range is as in part (a) and the hardness of each of ten randomly selected specimens is independently determined, what is the expected number of acceptable specimens among the ten?d. What is the probability that at most eight of ten independently selected specimens have a hardness of less than73.84? [Hint: Y = the number among the ten specimens with hardness less than 73.84 is a binomial variable; what is p?]

Answers

Answer:

a) The probability that a randomly chosen specimen has an acceptable hardness is 0.7938.

b) If the acceptable range of hardness is (70-c, 70+c), then the value of c would 95% of all specimens have an acceptable hardness of 5.88.

c) Expected number of acceptable specimens among the ten is 7.938.

d) Binomial with n = 10 and p = P(X < 73.84)

[tex]p = P(Z <(73.84 - 70) / 3 ) = P(Z < 1.28) = 0.8997\\\\P(X <= 8) = 1 - P(X = 9) - P(X = 10)\\= 0.2650635[/tex]

Step-by-step explanation:

a )

[tex]P(67 < X< 75) = P( (67 - 70) / 3 < X < (75 - 70) / 3 )\\\\= P( - 1 < Z < 1.67) = 0.9525 - 0.1587 = 0.7938[/tex]

b )

[tex]c = 1.96 * 3 = 5.88[/tex]                    { Since Z = 1.96 for 95% CI refer table.}

c )

Expected number of acceptable specimens among the ten [tex]= 10 * P(67 < X< 75) \\\\= 10 * 0.7938 = 7.938[/tex]

d )

Binomial with n = 10 and p = P(X < 73.84)

[tex]p = P(Z <(73.84 - 70) / 3 ) = P(Z < 1.28) = 0.8997\\\\P(X <= 8) = 1 - P(X = 9) - P(X = 10)\\= 0.2650635[/tex]

PLEASE HELP!! graph the circle whose equation is (x-6)^2 + (y+2)^2 =4

Answers

Answer:

Y= -x^2+12x-36

Step-by-step explanation:

Can anyone help me please ????

Answers

Hey there! The topic for this problem is Limit of Function!

As for the question, we are given the quadratic function and we have to find the limit, the value that approaches to a.

[tex] \large \boxed{lim_{x \longrightarrow a} f(x)}[/tex]

We call this, "The limit of f(x) when x approaches a."

Then you may ask, "How do we find the limit of function?". That is a very nice question! The answer to your problem is just substitute x-value in. Although this substitution method only applies when the approaching value doesn't make the denominator to 0. I believe that in the beginning of Limit topic, we learn how to find or evaluate the basic limit that only requires substitution.

So from the question, we receive:

[tex] \large{lim_{x \longrightarrow 2} ( {x}^{2} - 3x - 1)}[/tex]

Next step is to substitute x = 2 in the function.

[tex] \large{lim_{x \longrightarrow 2} ( {2}^{2} - 3(2) - 1)}[/tex]

Evaluate the value.

[tex] \large{lim_{x \longrightarrow 2} ( 4 - 6 - 1)} \\ \large{lim_{x \longrightarrow 2} ( - 3)}[/tex]

Cancel the limit out and there you have it!

[tex] \large \boxed{ - 3}[/tex]

Answer

The limit of quadratic function when x approaches 2 is -3.

Now whenever you learn limit, you must know that limit is when we substitute the approaching value. That means x —> 2 is not x = 2 but x approaches 2.

Regarding the limit, any questions and doubts can be asked through comment and I will get back to you soon!

Thank you for using Brainly and I hope you have a fantastic day! Good luck on the assignment.

Is triangle XYZ = ABC ? If so, name the postulate that applies. A. Congruent - ASA B. Congruent - SAS C. Might not be congruent D. Congruent - SSS

Answers

The answer is the SAS postulate. The two triangles have two pairs of corresponding, congruent sides, and the included angles are congruent. And thus, triangles XYZ and ABC are congruent by the SAS postulate.

Please help! What is the domain of the function shown in the graph?

Answers

Answer:

nnmb cgchmfxmgfmv vb  ,  mnio qrstuvwxyz

abcdefghijklmnoprstuvxxyz

Step-by-step explanation:

now i know my abcs next time wont you sing with me

A bag contains 6 black, 4 blue, and 8 white marbles. What is the probability that a marble drawn from the bag will be white?
a.1/18
b.2/5
c.4/9
d.4/5

Answers

Answer:

C.4/9

Step-by-step explanation:

You add the black marbles, blue marbles, and the white marbles together, which equal 18. Out of the 18 marbles, 8 of them are white so it would be 8/18.

but you have to simplify it. 8 and 18 have 2 as the greatest common factor. 8 divided by 2 is 4 and 18 divided by 2 is 9. Finally its 4/9 since you cant simplify it even more.

How tall is the table?
120cm
90cm
I

Answers

The table is 120 cm tall

The values of variables, such as the height of the table can be found by writing equations of their relationships

The height of the table is 105 cm

The reason the above height value is correct is as follows;

Known parameters:

The diagram shows a table, a cat and a mice

Let x, represent the height of the table, let y represent the height of the cat, and let z represent the height of the mice

From the given diagram, we have;

Height of the table + Height of the cat - Height of the mice  = 120 cm

x + y - z = 120...(1)

Height of the table + Height of the mice - Height of the cat   = 90 cm

x + z - y = 90...(2)

Adding equation (1) to equation (2) gives;

x + y - z + (x + z - y) = 120 + 90 = 210

x + y - z + (x + z - y) = 210

However;

x + y - z + (x + z - y) = x + x + y - y - z + z = 2·x

∴ x + y - z + (x + z - y) = 2·x = 210

x = 210/2 = 105

Therefore;

The height of the table, x = 105 cm

Learn more about word problems leading on simultaneous equations here:

https://brainly.com/question/16513646

Three bags contain 3 red, 7 black; 8 red, 2 black, and 4 red & 6 black balls
respectively. 1 of the bags is selected at random and a ball is drawn from it. If the ball
drawn is red, find the probability that it is drawn from the third bag.

Answers

Answer:

[tex]Probability = \frac{4}{15}[/tex]

Step-by-step explanation:

B1 = first bag

B2= second bag

B3 = third bag

Let A = ball drawn is red

Since, there are three bags.

Probability of choosing one bag=  P(B1) = P(B2) = P(B3) = 1/3.

From B1: Total balls = 10

3 red + 7 black balls.

Probability of drawing 1 red ball from it , P(A) = 3/10.

From B2: Total balls = 10

8 red + 2 black

Probability of drawing 1 red ball is, P(A) = 8/10

From B3 : Total Balls = 10

4 red + 6 black

Probability of drawing 1 red ball, P(A) = 4/10 .

To find Probability given that the ball drawn is red, that the ball is drawn from the third bag by Bayes' rule.

That is , P(B3|A)

                     [tex]=\frac{\frac{1}{3} \times \frac{4}{10}} { \frac{1}{3} \times \frac{3}{10} + \frac{1}{3} \times\frac{8}{10} + \frac{1}{3} \times \frac{4}{10}}[/tex]

   

                    [tex]=\frac{4}{30} \times \frac{30}{15}\\\\=\frac{4}{15}[/tex]

Therefore, the probability that it is drawn from the third bag is 4/15.

Answer:

4/15

Step-by-step explanation:

Solution of conditional probability problem:

Given:

Bags (3R,7B), (8R,2B), (4R,6B)

Let

P(R,i) = probability of drawing a red AND from bag i

P(R, 1) = 3/10 * (1/3) = 3/30

P(R, 2) = 8/10 * (1/3) = 8/30

P(R, 3) = 4/10 * (1/3) = 4/30

Let

Let P(R) = probability of drawing a red from any bag

P(R) = sum P(R,i)  for i = 1 to 3   using the addition rule

= 3/30 + 8/30 + 4/30

= 15/30

= 1 / 2

Conditional Probability of drawing from the third bag GIVEN that it is a red

= P(3 | R)

= P(R, 3) / P(R)

= 4/30 / (1/2)

= 8/30

= 4 / 15

(Since all bags contain 10 balls, by intuition, 4 red from third / 15 total red = 4/15)

6 Write 89.4945 correct to (a) nearest whole number, [1] (b) two decimal places. ​

Answers

Answer:

a)89

b)89.45

Step-by-step explanation:

A jacket costs $154.85. There is a 45% discount. What is the new price of the jacket.

A.) $68.68
B.) $85.17
C.) $224.53

Answers

b is is the best answer :) good luck

Answer:

B) $85,167

Step-by-step explanation:

u got discount 45% so u just have to pay 55% of it

cost = 55% x $154,85 = $85,1675

Intravenous fluid bags are filled by an automated filling machine. Assume that the fill volumes of the bags are independent, normal random variables with a standard deviation of 0.08 fluid ounces.
(a)What is the standard deviation of the average fill volume of 22 bags?
(b)The mean fill volume of the machine is 6.16 ounces, what is the probability that the average fill volume of 22 bags is below 5.95 ounces?
(c)What should the mean fill volume equal in order that the probability that the average of 22 bags is below 6.1 ounces is 0.001?

Answers

Answer:

a) 0.0171 fluid ounces.

b) 0% probability that the average fill volume of 22 bags is below 5.95 ounces

c) The mean should be of 6.153 fluid ounces.

Step-by-step explanation:

To solve this question, we need to understand the normal probability distribution and the central limit theorem.

Normal Probability Distribution:

Problems of normal distributions can be solved using the z-score formula.

In a set with mean [tex]\mu[/tex] and standard deviation [tex]\sigma[/tex], the z-score of a measure X is given by:

[tex]Z = \frac{X - \mu}{\sigma}[/tex]

The Z-score measures how many standard deviations the measure is from the mean. After finding the Z-score, we look at the z-score table and find the p-value associated with this z-score. This p-value is the probability that the value of the measure is smaller than X, that is, the percentile of X. Subtracting 1 by the p-value, we get the probability that the value of the measure is greater than X.

Central Limit Theorem

The Central Limit Theorem establishes that, for a normally distributed random variable X, with mean [tex]\mu[/tex] and standard deviation [tex]\sigma[/tex], the sampling distribution of the sample means with size n can be approximated to a normal distribution with mean [tex]\mu[/tex] and standard deviation [tex]s = \frac{\sigma}{\sqrt{n}}[/tex].

For a skewed variable, the Central Limit Theorem can also be applied, as long as n is at least 30.

Standard deviation of 0.08 fluid ounces.

This means that [tex]\sigma = 0.08[/tex]

(a)What is the standard deviation of the average fill volume of 22 bags?

This is s when n = 22. So

[tex]s = \frac{\sigma}{\sqrt{n}}[/tex]

[tex]s = \frac{0.08}{\sqrt{22}}[/tex]

[tex]s = 0.0171[/tex]

(b)The mean fill volume of the machine is 6.16 ounces, what is the probability that the average fill volume of 22 bags is below 5.95 ounces?

We have that [tex]\mu = 6.16[/tex]. The probability is the p-value of Z when X = 5.95. So

[tex]Z = \frac{X - \mu}{\sigma}[/tex]

By the Central Limit Theorem

[tex]Z = \frac{X - \mu}{s}[/tex]

[tex]Z = \frac{5.95 - 6.16}{0.0171}[/tex]

[tex]Z = -12.3[/tex]

[tex]Z = -12.3[/tex] has a p-value of 0.

0% probability that the average fill volume of 22 bags is below 5.95 ounces.

(c)What should the mean fill volume equal in order that the probability that the average of 22 bags is below 6.1 ounces is 0.001?

[tex]X = 6.1[/tex] should mean that Z has a p-value of 0.001, so Z = -3.09. Thus

[tex]Z = \frac{X - \mu}{s}[/tex]

[tex]-3.09 = \frac{6.1 - \mu}{0.0171}[/tex]

[tex]6.1 - \mu = -3.09*0.0171[/tex]

[tex]\mu = 6.153[/tex]

The mean should be of 6.153 fluid ounces.

What is the inverse function of y = 2x - 8

Answers

Answer:

Step-by-step explanation:

y = 2x-8

2x = y+8

x = 0.5y+4

inverse function: y = 0.5x+4

which of the following statements is true

Answers

Answer: B ACE is similar to DCB

Step-by-step explanation:

Find the percent of decrease from 46 songs to 41 songs. Round to the nearest tenth of a percent if necessary.
percent of decrease
%

Answers

Answer:

10.9 %

Step-by-step explanation:

46 - 41 = 5

5/46 * 100% = 10.8695652174%

Rounded

10.9 %

What is the slope of the line represented by the equation f(x) = -3x + 7?

A -7
B -3
C 3
D 7

Answers

Answer:

-3

Step-by-step explanation:

in these equations the slope is multiplied by x which is -3 in this one.

Answer:

[tex]-3[/tex]

Step-by-step explanation:

The equation of a line is [tex]y=mx+b[/tex], and m is the gradient which is the slope. Hence, the number with x is the slope, which is [tex]-3[/tex].

Hope that helped.

HELP PLEASE MATH PROBLEM

Answers

Answer:

x=41

Step-by-step explanation:

LM =JM

154=4x-10

154+10=4x

164=4x

164/4=4x/4

41=x

hope this is helpful

the third option , x=41 !!

Find the Diameter of the circle, whose radius is 17 cm.

Answers

Answer:

34 cm

Step-by-step explanation:

The radius is half of the diameter, so 17 cm is half of 34 cm.

Diameter = 34 cm

When we expand (2x + 1/2)^6, what is the coefficient on the x^4 term?

Answers

Answer: The coefficient before x^4 is 60

Step-by-step explanation:

Hey! So I am not an expert at this, but you have to use the binomial theorem

I have attached of the Pascals Triangle (one shows the row numbering as well)

Basically in a pascal triangle, you add the two numbers above it to get the next number below

As you can see, the rows start from 0 instead of 1

The 6th row contains the numbers 1, 6, 15, 20, 15, 6, 1 which would be the coefficient terms

NOTE: the exponents always add to 6, the first term starts at 6 and decrease it's exponent by 1 each time (6, 5, 4, 3, 2, 1, 0) and the second term increases it's exponent by 1 each time (0, 1, 2, 3, 4, 5, 6)

Using this information the third term from the sixth row (15) would be where it is x^4 (I have circled it on the second image)

It would be 15 × 2^4 × (1/2)^2 = 60

The reason why it is 2^4 and (1/2)^2 is because the third term has the exponents 4 and 2 (bolded on the NOTE) which means that the first term must be put to the power of 4 and the second term must be put to the 2nd power

Sorry for the lousy explanation. I really hope this makes sense! Let me know if this helped :)

The product of three consecutive numbers is divisible by ​

Answers

Answer:

6

Step-by-step explanation:

The product of three consecutive numbers is divisible by ​6

Let us say the numbers are x, x+1 , x+2

if x = 1,

Product of the three consecutive numbers,

(1)(2)(3)

=> 6, which is divisible by 6

if x = 2,

Product of the three consecutive numbers,

(2)(3)(4)

=> 24, which is divisible by 6

Similarly if we take any 3 consecutive numbers their product will be divisible by 6.

Q23. Find the value of k, if x = 2, y = 1 is a solution of the equation 2x + 3y = k.​

Answers

Answer:

k=7

Step-by-step explanation:

2x+3y=k

2(2)+3(1)=k

4+3=k

k=7

Answer:

7.

Step-by-step explanation:

Substitute x = 2 and y = 1 into the given equation:

2(2) + 3(1) = k

4 + 3 = k

k = 7.

course
Look at the following number line:
- 10
-5
0
5
10
What are two ways to write the inequality graphed?
x>-1 and -1 XS-1 and -12X
x < -1 and -1 > X
x2-1 and -1 5x

Answers

first and last one i think

One evening Papa John’s sold a total of 33 pizzas topped with pepperoni, sausage, or pepperoni and sausage. There were 29 pizzas that had pepperoni. Of these, 15 also had sausage. How many more pizzas had pepperoni only than had sausage only?

Answers

Answer:

10

Step-by-step explanation:

Total pizza topped with pepperoni, sausage or pepperoni and sausage = 33

Number of pizzas with pepperoni = 29

Number of pizzas with pepperoni and sausage = 15

Pizza with pepperoni only = 29 - 15 = 14

Pizza with sausage only = 33 - 29 = 4

Pepperoni only than sausage only :

14 - 4 = 10

Other Questions
A solution of ammonia NH3(aq) is at equilibrium. How would the equilibriumchange if NH4+ were added to the solution? An item costs $370 before tax and the sales tax is $11.10. Find the sales tax rate::Write your answer as a percentage. Algebra 2 Problem! Rational Roots Which statement best describes a command economy?Government intervention in economic choices is strictly forbidden.The government determines economic choices and makes most decisions.The decisions made by producers and consumers drive all economic choices.Producers and consumers make some economic choices while the government makes others. Which of the following is the solution set for -3t + 11 > 20?t < 3t > 3t < -3t > -3 Read the passage from "The Tell-Tale Heart.I think it was his eye! yes, it was this! He had the eye of a vulturea pale blue eye, with a film over it. Whenever it fell upon me, my blood ran cold; and so by degreesvery graduallyI made up my mind to take the life of the old man, and thus rid myself of the eye forever. Now this is the point. You fancy me mad. Madmen know nothing. But you should have seen me. You should have seen how wisely I proceededwith what cautionwith what foresightwith what dissimulation I went to work! I was never kinder to the old man than during the whole week before I killed him.How is suspense created for the reader in this passage?The reader wonders where the old man lives.The reader is waiting to find out details of the murder.The reader envisions a vulture with pale blue eyes.The reader does not know why the old man is killed. PLZZZZZZZZ HELPPPPPP Your money will be refunded prove to be .. (satisfy) Whats the answer to the question? Which 2 options are correct? what event happened immediately following the election of lincoln in 1860? what part did pragmatism and deism play in the role of the united states of america? Describe the immigration trend between 1881 and 19207. What Factors contributed to this trend? Consider the factors that affect muscular strength. Read each scenario and then identify each label into the appropriate category based on whether each would result in a stronger or weaker muscle contraction.a. Potassium accumulate in the sarcoplasmb. Increase in muscle belly circumferencec. Lesser proportion of motor neurons to muscle fibresd. Begin contractions with muscle already 50% contractione. Lower sarcoplasm pHf. Increased stimulus frequency.g. Circular arrangement of muscle fasciclesh. increased requirement. What is the value of x in the equation 6 +4x = 7 What is the equation of the line that passes through the point (-7, -4) and has aslope of 1? A partially inflated balloon is faded over the open end of a glass beaker that contains water the beaker is placed in an ice bath which cause the water and causes the balloon to the flea what are the correct designs for the heat and work changes to the beaker and balloon system which of the following items has the most inertia while at rest 32The angles shown are supplementary. What is the value of x? 0(2 Points)(5x595O 180 degree 17 degreeO 22 degree45 degreeHELPP I NEED THIS ASAP ITS FOR MY FINALSS Using the following equation how many grams of water you would get from 886 g of glucose:C6H12O6 + 6O2 6CO2 + 6H2O + energy